1answer.
Ask question
Login Signup
Ask question
All categories
  • English
  • Mathematics
  • Social Studies
  • Business
  • History
  • Health
  • Geography
  • Biology
  • Physics
  • Chemistry
  • Computers and Technology
  • Arts
  • World Languages
  • Spanish
  • French
  • German
  • Advanced Placement (AP)
  • SAT
  • Medicine
  • Law
  • Engineering
sattari [20]
3 years ago
5

Hello, Brainly community!

Mathematics
1 answer:
ioda3 years ago
8 0

Answer:

(B)  \displaystyle \frac{W(3.1) - W(2.9)}{0.2}

General Formulas and Concepts:

<u>Calculus</u>

Limits

Derivatives

  • The definition of a derivative is the slope of the tangent line.

Derivative Notation

Instantaneous Rates

  • Tangent Line: \displaystyle f'(x) = \frac{f(b) - f(a)}{b - a}

Step-by-step explanation:

Since we are trying to find a <em>rate</em> at which W(t) changes, we must find the <em>derivative</em> at <em>t</em> = 3.

We are given 2 close answer choices that would have the same <em>numerical</em> answer but different <em>meanings</em>:

  1. (A)  \displaystyle  \lim_{t \to 3} W(t)
  2. (B)  \displaystyle \frac{W(3.1) - W(2.9)}{0.2}

If we look at answer choice (A), we see that our units would simply just be volume. It would not have the units of a rate of change. Yes, it may be the closest numerically correct answer, but it does not tell us the <em>rate</em> at which the volume would be changing and it is not a derivative.

If we look at answer choice (B), we see that our units would be cm³/s, and that is most certainly a rate of change. Answer choice (B) is also a <em>derivative</em> at <em>t</em> = 3, and a derivative tells us what <em>rate</em> something is changing.

∴ Answer choice (B) will give us the best estimate for the value of the instantaneous rate of change of W(t) when <em>t</em> = 3.

Topic: AP Calculus AB/BC (Calculus I/I + II)

Unit: Differentiation

Book: College Calculus 10e

You might be interested in
Write the equation of a line with a slope of 3 and a y-intercept of 1.
noname [10]

Take a look at the image below.

6 0
3 years ago
Find the distance between the points (-3,2) and (0,3).
Alex777 [14]

Answer:

The distance between (-3, 2) and (0,3) is √10.

Step-by-step explanation:

As we go from (-3,2)  to  (0,3), x increases by 3 and y increases by 1.

Think of a triangle with base 3 and height 1.  Use the Pythagorean Theorem to find the length of the hypotenuse, which represents the distance between the points (-3, 2) and (0, 3):

distance = √(3² + 1²) = √10

The distance between (-3, 2) and (0,3) is √10.

7 0
3 years ago
Read 2 more answers
Solve the following equations using any method. Show your work and check your
Gemiola [76]

a) The solution for the equation 2x + 16 = 5x + 4  is x=4.

b) The solution for the equation 3x − 5 = 2x + 14 is x=19.

c) The solution for the equation 5x − 5 = x + 15 is x=5.

<u>Step-by-step explanation:</u>

<u>a) The given equation is  2x + 16 = 5x + 4 </u>

  • From the above equation, it can be found that the equation consists of x terms and constant terms.
  • Therefore, bring all the x terms on one side and the constant terms on other side of the equation.
  • This gives the simplified form of the equation to solve for x value.

⇒ 16-4 = 5x-2x

⇒ 12 = 3x

⇒ x = 12/3

⇒ x = 4

The solution for the given equation is x=4.

<u>To check, substitute x=4 in the equation :</u>

2(4) + 16 = 5(4) + 4

8+16 = 20+4

24 = 24

Therefore, the solution x=4 satisfies the equation 2x + 16 = 5x + 4

Similarly,

<u>b) The given equation is 3x − 5 = 2x + 14</u>

⇒ 3x-2x = 14+5

⇒ x = 19

The solution for the given equation is x=19.

<u>To check, substitute x=19 in the equation :</u>

3(19) - 5 = 2(19) + 14

57-16 = 38+14

52 = 52

Therefore, the solution x=19 satisfies the equation 3x − 5 = 2x + 14

Similarly,

<u>c) The given equation is 5x − 5 = x + 15</u>

⇒ 5x-x = 15+5

⇒ 4x = 20

⇒ x = 20/4

⇒ x = 5

The solution for the given equation is x=5.

<u>To check, substitute x=5 in the equation :</u>

5(5) - 5 = 5 + 15

25-5 = 20

20 = 20

Therefore, the solution x=5 satisfies the equation 5x − 5 = x + 15

5 0
3 years ago
What additional information is need
Gelneren [198K]
Umm can you be more specific on your question so I can help you need to explain what you need help on
7 0
3 years ago
Assume that your present job pays a monthly gross salary of $1,560. You are offered a new position that pays $8.60 per hour with
motikmotik

Answer:

There are 4 weeks in a month and 40 working hours in a week

Since the $8.60 per hour, the gross monthly salary for your new position = 8.60*40*4 = $1,376

Your gross monthly salary at present job = $1,560

So the additional amount that you need to earn at your new position = 1,560-1,376 = $184

The hourly wage for addtional hours more than 40 hours at the new position = 1 1/2 * 8.60 = 1.5*8.60 = $12.90

So, number of hours that you need to work extra = 184/12.90 = 14.26 hours per month

So, number of hours that you need to work extra per week = 14.26/4 =  3.566 hours or 3.57 hours (Rounded to 2 decimals)

Step-by-step explanation:

4 0
3 years ago
Other questions:
  • how many times smaller is the 8 in 3,810,200 than the 8 in 8,911,053? answer options with 5 options a. 1,000 times smaller b. 10
    7·1 answer
  • When dividing 6,000,000 by a power of ten, the digits in the quotient moved 3 places to the right. What was the power of ten?
    8·1 answer
  • What number could replace the □?
    12·2 answers
  • On a scale drawing, the scale is 1 inch = 1 foot. Find the scale factor.
    10·2 answers
  • A semi trailer is being packed with boxes that are 1 1/2 feet on each edge. The cargo space of the trailer is 28 feet long, 8 1/
    11·1 answer
  • When a number x is multiplied by 9, the result is 35 .<br><br> What is the value of x?
    14·1 answer
  • om invests $6249 in an account that pays 4.15% interest per year for 4 years. How much simple interest will he earn? Round your
    7·1 answer
  • Simplify the expression 53 + 3(5 − 3). 137 131 27 21
    8·1 answer
  • Solve for y.<br> (3x - 3)<br> Enter your answer in the box.<br> y=<br> C<br> (8-4)
    9·1 answer
  • Can someone help this for me?
    13·1 answer
Add answer
Login
Not registered? Fast signup
Signup
Login Signup
Ask question!